Home / IBDP Maths SL 4.1 Concepts of population, sample AA HL Paper 1- Exam Style Questions

IBDP Maths SL 4.1 Concepts of population, sample AA HL Paper 1- Exam Style Questions

IBDP Maths SL 4.1 Concepts of population, sample AA HL Paper 1- Exam Style Questions- New Syllabus

Question

On a Monday at an amusement park, a sample of 40 visitors was randomly selected as they were leaving the park. They were asked how many times that day they had been on a ride called The Dragon. This information is summarized in the following frequency table.

Frequency table

It can be assumed that this sample is representative of all visitors to the park for the following day.

(a) For the following day, Tuesday, estimate:

(i) the probability that a randomly selected visitor will ride The Dragon;

(ii) the expected number of times a visitor will ride The Dragon.

It is known that 1000 visitors will attend the amusement park on Tuesday. The Dragon can carry a maximum of 10 people each time it runs.

(b) Estimate the minimum number of times The Dragon must run to satisfy demand.

▶️ Answer/Explanation
Solution (a)(i)

To estimate the probability that a visitor will ride The Dragon:

  1. Total visitors sampled = 40
  2. Visitors who rode at least once:
    • 16 rode once
    • 13 rode twice
    • 2 rode three times
    • 3 rode four times
  3. Total riders = 16 + 13 + 2 + 3 = 34

Probability calculation:

\[ P(\text{riding}) = \frac{34}{40} = \frac{17}{20} = 0.85 \]

Thus, there’s an 85% probability a visitor will ride The Dragon.

Solution (a)(ii)

To find the expected number of rides per visitor:

\[ E(X) = \sum [x \times P(X=x)] \]

Calculating each term:

Rides (x)FrequencyP(X=x)x × P(X=x)
066/400
11616/4016/40
21313/4026/40
322/406/40
433/4012/40

Summing the contributions:

\[ E(X) = 0 + \frac{16}{40} + \frac{26}{40} + \frac{6}{40} + \frac{12}{40} = \frac{60}{40} = 1.5 \]

Therefore, the expected number of rides per visitor is 1.5.

Solution (b)

To estimate the minimum number of runs needed:

  1. Calculate total expected rides:

    \[ 1.5 \text{ rides/visitor} \times 1000 \text{ visitors} = 1500 \text{ rides} \]

  2. Determine runs required:

    The Dragon carries 10 people per run, so:

    \[ \frac{1500 \text{ rides}}{10 \text{ rides/run}} = 150 \text{ runs} \]

However, we must consider that:

  • This is the minimum estimate based on averages
  • Actual demand may fluctuate due to random variations
  • The park might want to add a safety margin for peak times

Thus, while 150 runs is the theoretical minimum, practical operation might require slightly more.

Question

The discrete random variable \( X \) has the following probability distribution, where \( p \) is a constant.

\( x \)\( 0 \)\( 1 \)\( 2 \)\( 3 \)\( 4 \)
\( P(X=x) \)\( p \)\( 0.5-p \)\( 0.25 \)\( 0.125 \)\( p^3 \)

(a) Find the value of \( p \). \([2]\)

(b)(i) Find \( \mu \), the expected value of \( X \). \([2]\)

(b)(ii) Find \( P(X > \mu) \). \([2]\)

▶️ Answer/Explanation
Solution (a)

For any probability distribution, the sum of all probabilities must equal \( 1 \):

\[ p + (0.5 – p) + 0.25 + 0.125 + p^3 = 1 \]

Simplify the equation:

\[ 0.5 + 0.25 + 0.125 + p^3 = 1 \]
\[ 0.875 + p^3 = 1 \]
\[ p^3 = 0.125 \]

Solve for \( p \):

\[ p = \sqrt[3]{0.125} = 0.5 \]

Verification: Substituting back:
\( P(X=0) = 0.5 \)
\( P(X=1) = 0.5 – 0.5 = 0 \)
\( P(X=4) = (0.5)^3 = 0.125 \)
Sum \( = 0.5 + 0 + 0.25 + 0.125 + 0.125 = 1 \) ✓

Solution (b)(i)

The expected value \( \mu \) is calculated as:

\[ \mu = \sum [x \times P(X=x)] \]

Compute each term:

\[ = (0 \times 0.5) + (1 \times 0) + (2 \times 0.25) + (3 \times 0.125) + (4 \times 0.125) \]
\[ = 0 + 0 + 0.5 + 0.375 + 0.5 \]
\[ = 1.375 \text{ or } \frac{11}{8} \]

Thus, the expected value is \( \boxed{1.375} \).

Solution (b)(ii)

To find \( P(X > \mu) \) where \( \mu = 1.375 \):

We consider values of \( X \) greater than \( 1.375 \), which are \( X = 2, 3, \) and \( 4 \).

Calculate the probability:

\[ P(X > 1.375) = P(X=2) + P(X=3) + P(X=4) \]
\[ = 0.25 + 0.125 + 0.125 \]
\[ = 0.5 \]

Thus, the probability is \( \boxed{0.5} \).

Mark Scheme

(a) \([2]\)

  • M1: Correct equation setup \( (\sum P(X=x) = 1) \)
  • A1: Correct solution \( (p = 0.5) \)

(b)(i) \([2]\)

  • M1: Correct expected value formula application
  • A1: Correct final answer \( (1.375 \text{ or } \frac{11}{8}) \)

(b)(ii) \([2]\)

  • M1: Correct identification of \( P(X > \mu) \) cases
  • A1: Correct final answer \( (0.5) \)

Total: 6 marks

✅ Key Concepts:
  1. Probability Distributions: \( \sum P(X=x) = 1 \)
  2. Expected Value: \( \mu = E[X] = \sum x \cdot P(X=x) \)
  3. Cumulative Probability: \( P(X > a) = \sum_{x > a} P(X=x) \)
  4. Algebraic Solutions: Solving \( p^3 = 0.125 \)
Question

Consider the data \( x_1, x_2, x_3, \ldots, x_n \), with mean \( \overline{x} \), and standard deviation \( s \).

  1. If each number is increased by \( k \):
    1. Show that the new mean is \( \overline{x} + k \) (i.e., it is also increased by \( k \))
    2. Show that the new standard deviation is \( s \) (i.e., it remains the same)
  2. If each number is multiplied by \( k \):
    1. Show that the new mean is \( k\overline{x} \) (i.e., it is also multiplied by \( k \)
    2. Show that the new standard deviation is \( ks \) (i.e., it is also multiplied by \( k \)
    3. Write down the relation between the original and the new variance
▶️ Answer/Explanation
Solution (a)(i)

New mean calculation:

\[ \overline{x}_{\text{new}} = \frac{\sum_{i=1}^{n}(x_i + k)}{n} = \frac{\sum_{i=1}^{n}x_i + \sum_{i=1}^{n}k}{n} = \frac{\sum_{i=1}^{n}x_i}{n} + \frac{nk}{n} = \overline{x} + k \]

Thus, the new mean is \( \boxed{\overline{x} + k} \).

Solution (a)(ii)

New standard deviation calculation:

\[ s_{\text{new}} = \sqrt{\frac{\sum_{i=1}^{n}[(x_i + k) – (\overline{x} + k)]^2}{n}} = \sqrt{\frac{\sum_{i=1}^{n}(x_i – \overline{x})^2}{n}} = s \]

Thus, the standard deviation remains \( \boxed{s} \).

Solution (b)(i)

New mean calculation:

\[ \overline{x}_{\text{new}} = \frac{\sum_{i=1}^{n}(kx_i)}{n} = \frac{k\sum_{i=1}^{n}x_i}{n} = k\overline{x} \]

Thus, the new mean is \( \boxed{k\overline{x}} \).

Solution (b)(ii)

New standard deviation calculation:

\[ s_{\text{new}} = \sqrt{\frac{\sum_{i=1}^{n}(kx_i – k\overline{x})^2}{n}} = \sqrt{\frac{k^2\sum_{i=1}^{n}(x_i – \overline{x})^2}{n}} = k\sqrt{\frac{\sum_{i=1}^{n}(x_i – \overline{x})^2}{n}} = ks \]

Thus, the new standard deviation is \( \boxed{ks} \).

Solution (b)(iii)

Variance relationship:

Since \( s_{\text{new}} = ks \), then:

\[ s_{\text{new}}^2 = (ks)^2 = k^2s^2 \]

Thus, the new variance is \( \boxed{k^2} \) times the original variance.

Mark Scheme

(a)(i) [2 marks]

  • M1: Correct expansion of sum
  • A1: Correct final result \( (\overline{x} + k) \)

(a)(ii) [2 marks]

  • M1: Correct simplification inside square root
  • A1: Correct conclusion \( (s) \)

(b)(i) [2 marks]

  • M1: Correct factorization of \( k \)
  • A1: Correct final result \( (k\overline{x}) \)

(b)(ii) [2 marks]

  • M1: Correct factorization of \( k^2 \)
  • A1: Correct final result \( (ks) \)

(b)(iii) [1 mark]

  • A1: Correct variance relationship \( (k^2) \)

Total: 9 marks

✅ Key Concepts:
  1. Mean Transformation: Additive \( (+k) \) vs multiplicative \( (\times k) \) changes
  2. Standard Deviation: Invariant under addition, scales linearly under multiplication
  3. Variance: Scales quadratically \( (k^2) \) under multiplication
  4. Summation Properties: Linearity of summation operations
Question

Consider the following data

\( x \)1234
\( y \)2378
  1. Find the mean and the variance for the values of \( x \).
  2. Find the mean and the variance for the values of \( y \).
  3. Find the correlation coefficient \( r \).
  4. Describe the relation between \( x \) and \( y \).
  5. Find the equation \( y = ax+b \) of the regression line for \( y \) on \( x \).
  6. Find the equation \( x = cy+d \) of the regression line for \( x \) on \( y \).
  7. Find the inverse of the function in question (e); Is it the function in question (f)?
▶️ Answer/Explanation
Solution (a)

Mean (\( \mu_x \)):

\[ \mu_x = \frac{1+2+3+4}{4} = \frac{10}{4} = 2.5 \]

Variance (\( \sigma_x^2 \)):

\[ \sigma_x^2 = \frac{(1-2.5)^2 + (2-2.5)^2 + (3-2.5)^2 + (4-2.5)^2}{4} \]

\[ = \frac{2.25 + 0.25 + 0.25 + 2.25}{4} = \frac{5}{4} = 1.25 \]

Standard deviation: \( \sigma_x = \sqrt{1.25} \approx 1.11803 \)

\(\boxed{\mu_x = 2.5,\ \sigma_x^2 = 1.25}\)

Solution (b)

Mean (\( \mu_y \)):

\[ \mu_y = \frac{2+3+7+8}{4} = \frac{20}{4} = 5 \]

Variance (\( \sigma_y^2 \)):

\[ \sigma_y^2 = \frac{(2-5)^2 + (3-5)^2 + (7-5)^2 + (8-5)^2}{4} \]

\[ = \frac{9 + 4 + 4 + 9}{4} = \frac{26}{4} = 6.5 \]

Standard deviation: \( \sigma_y = \sqrt{6.5} \approx 2.54951 \)

\(\boxed{\mu_y = 5,\ \sigma_y^2 = 6.5}\)

Solution (c)

Correlation coefficient (\( r \)):

First calculate covariance:

\[ \text{Cov}(x,y) = \frac{(1-2.5)(2-5) + (2-2.5)(3-5) + (3-2.5)(7-5) + (4-2.5)(8-5)}{4} \]

\[ = \frac{4.5 + 1 + 1 + 4.5}{4} = \frac{11}{4} = 2.75 \]

Then:

\[ r = \frac{\text{Cov}(x,y)}{\sigma_x \sigma_y} = \frac{2.75}{(1.11803)(2.54951)} \approx 0.965 \]

\(\boxed{r \approx 0.965}\)

Solution (d)

The correlation coefficient \( r \approx 0.965 \) indicates a:

\(\boxed{\text{Strong positive linear relationship between } x \text{ and } y}\)

(Since \( r \) is close to 1 and positive)

Solution (e)

Regression line \( y \) on \( x \):

Slope (\( a \)):

\[ a = r \cdot \frac{\sigma_y}{\sigma_x} = 0.965 \cdot \frac{2.54951}{1.11803} \approx 2.2 \]

Intercept (\( b \)):

\[ b = \mu_y – a\mu_x = 5 – (2.2)(2.5) = -0.5 \]

Equation:

\(\boxed{y = 2.2x – 0.5}\)

Solution (f)

Regression line \( x \) on \( y \):

Slope (\( c \)):

\[ c = r \cdot \frac{\sigma_x}{\sigma_y} = 0.965 \cdot \frac{1.11803}{2.54951} \approx 0.423 \]

Intercept (\( d \)):

\[ d = \mu_x – c\mu_y = 2.5 – (0.423)(5) \approx 0.385 \]

Equation:

\(\boxed{x = 0.423y + 0.385}\)

Solution (g)

Inverse of (e):

Starting with \( y = 2.2x – 0.5 \):

\[ y + 0.5 = 2.2x \]

\[ x = \frac{y + 0.5}{2.2} = \frac{y}{2.2} + \frac{0.5}{2.2} \]

\[ x \approx 0.455y + 0.227 \]

Comparison with (f):

The inverse \( x \approx 0.455y + 0.227 \) is different from the regression \( x = 0.423y + 0.385 \) found in (f).

\(\boxed{\text{No, they are different functions}}\)

Scroll to Top